Difference between revisions of "2000 AMC 10 Problems/Problem 16"
5849206328x (talk | contribs) (→Solution) |
5849206328x (talk | contribs) (→Solution) |
||
Line 68: | Line 68: | ||
which is answer choice <math>\boxed{\text{B}}</math> | which is answer choice <math>\boxed{\text{B}}</math> | ||
+ | |||
+ | ===Solution 2=== | ||
+ | |||
+ | <asy> | ||
+ | path seg1, seg2; | ||
+ | seg1=(6,0)--(0,3); | ||
+ | seg2=(2,0)--(4,2); | ||
+ | dot((0,0)); | ||
+ | dot((1,0)); | ||
+ | fill(circle((2,0),0.1),black); | ||
+ | dot((3,0)); | ||
+ | dot((4,0)); | ||
+ | dot((5,0)); | ||
+ | fill(circle((6,0),0.1),black); | ||
+ | dot((0,1)); | ||
+ | dot((1,1)); | ||
+ | dot((2,1)); | ||
+ | dot((3,1)); | ||
+ | dot((4,1)); | ||
+ | dot((5,1)); | ||
+ | dot((6,1)); | ||
+ | dot((0,2)); | ||
+ | dot((1,2)); | ||
+ | dot((2,2)); | ||
+ | dot((3,2)); | ||
+ | fill(circle((4,2),0.1),black); | ||
+ | dot((5,2)); | ||
+ | dot((6,2)); | ||
+ | fill(circle((0,3),0.1),black); | ||
+ | dot((1,3)); | ||
+ | dot((2,3)); | ||
+ | dot((3,3)); | ||
+ | dot((4,3)); | ||
+ | dot((5,3)); | ||
+ | dot((6,3)); | ||
+ | draw(seg1); | ||
+ | draw(seg2); | ||
+ | pair [] x=intersectionpoints(seg1,seg2); | ||
+ | fill(circle(x[0],0.1),black); | ||
+ | label("$A$",(0,3),NW); | ||
+ | label("$B$",(6,0),SE); | ||
+ | label("$C$",(4,2),NE); | ||
+ | label("$D$",(2,0),S); | ||
+ | label("$E$",x[0],N); | ||
+ | label("$F$",(2.5,.5),E); | ||
+ | draw((6,0)--(4,2)); | ||
+ | draw((0,3)--(2.5,.5)); | ||
+ | </asy> | ||
+ | |||
+ | Draw the perpendiculars from <math>A</math> and <math>B</math> to <math>CD</math>, respectively. As it turns out, <math>BC \perp CD</math>. Let <math>F</math> be the point on <math>CD</math> for which <math>AF\perp CD</math>. | ||
+ | |||
+ | <math>m\angle AFE=m\angle BCE=90^\circ</math>, and <math>m\angle AEF=m\angle CEB</math>, so by AA similarity, <cmath>\triangle AFE\sim \triangle BCE \Rightarrow \frac{AF}{AE}=\frac{BC}{BE}</cmath> | ||
+ | |||
+ | By the Pythagorean Theorem, we have <math>AB=\sqrt{3^2+6^2}=3\sqrt{5}</math>, <math>AF=\sqrt{2.5^2+2.5^2}=2.5\sqrt{2}</math>, and <math>BC=\sqrt{2^2+2^2}+2\sqrt{2}</math>. Let <math>AE=x</math>, so <math>BE=3\sqrt{5}-x</math>, then <cmath>\frac{2.5\sqrt{2}}{x}=\frac{2\sqrt{2}}{3\sqrt{5}-x}</cmath> <cmath>x=\frac{5\sqrt{5}}{3}</cmath> | ||
+ | |||
+ | This is answer choice <math>\boxed{\text{B}}</math> | ||
==See Also== | ==See Also== | ||
{{AMC10 box|year=2000|num-b=15|num-a=17}} | {{AMC10 box|year=2000|num-b=15|num-a=17}} |
Revision as of 07:33, 11 January 2009
Problem
The diagram shows lattice points, each one unit from its nearest neighbors. Segment meets segment at . Find the length of segment .
Solution
Solution 1
Let be the line containing and and let be the line containing and . If we set the bottom left point at , then , , , and .
The line is given by the equation . The -intercept is , so . We are given two points on , hence we can compute the slope, to be , so is the line
Similarly, is given by . The slope in this case is , so . Plugging in the point gives us , so is the line .
At , the intersection point, both of the equations must be true, so
We have the coordinates of and , so we can use the distance formula here:
which is answer choice
Solution 2
Draw the perpendiculars from and to , respectively. As it turns out, . Let be the point on for which .
, and , so by AA similarity,
By the Pythagorean Theorem, we have , , and . Let , so , then
This is answer choice
See Also
2000 AMC 10 (Problems • Answer Key • Resources) | ||
Preceded by Problem 15 |
Followed by Problem 17 | |
1 • 2 • 3 • 4 • 5 • 6 • 7 • 8 • 9 • 10 • 11 • 12 • 13 • 14 • 15 • 16 • 17 • 18 • 19 • 20 • 21 • 22 • 23 • 24 • 25 | ||
All AMC 10 Problems and Solutions |